Solve the system using multiplication for the linear combination method.
6x – 3y = 3
-2x + 6y = 14
What is the solution to the system?

Answers

Answer 1
6x-3y=3
-2x+6y=14.
First step is multiply the top equation by 2, so you have
12x-6y=6
-2x+6y=14
-6y and +6y cancel each other out, so if you simplify, you have
10x=20, which is then solved by dividing each side by 10, and getting x=2.

Now plug x=2 into one of the equations (I chose the first)
6(2)-3y=3
12-3y=3
-3y=-9, divide each side by -3 and get y=3.
so your point is (2,3)
Answer 2

Answer:

(2, 3)

Step-by-step explanation:


Related Questions

In a fruit cocktail, for every 15 ml of orange juice you need 5 ml of apple juice and 50 ml of coconut milk. What is the ratio of apple juice to coconut milk to orange juice expressed in its simplest form?

Answers

Answer:

1:10:3

Step-by-step explanation:

Dan bought x pounds of potatoes for $0.85 per pound and y pounds of grapes for $1.29 per pound. The total cost was less than $5. Which inequality represents his purchase?

1.29x + 0.85y < 5
1.29x + 0.85y > 5
0.85x + 1.29y > 5
0.85x + 1.29y < 5

Answers

Answer:

it's D

Step-by-step explanation:

The inequality that represents Dan's purchase is 0.85x + 1.29y < 5.

The correct option is D.

What is inequality?

In mathematics, "inequality" refers to a relationship between two expressions or values that are not equal to each other. To solve the inequality, you may multiply or divide each side by the same positive number, add the same amount to each side, take the same amount away from each side, and more. You must flip the inequality sign if you multiply or divide either side by a negative number.

The cost of x pounds of potatoes at $0.85 per pound is 0.85x dollars.

The cost of y pounds of grapes at $1.29 per pound is 1.29y dollars.

The total cost of the purchase is the sum of these costs, which is:

0.85x + 1.29y

The inequality that represents the total cost being less than $5 is:

0.85x + 1.29y < 5

So, the correct answer is 0.85x + 1.29y < 5.

To learn more about inequality;

brainly.com/question/28823603

#SPJ7

Which number belongs to the solution set of the equation?
x + 3 = 45
A. 15
B. 48
C. 43
D. 42

Answers

Answer:

D

Step-by-step explana

[tex] \sf \: The \: value \: of \: x \: is \: the \: number \: that \: belongs \: to \: the \: solution \: \\ \sf \: set \: of \: the \: equation. \: So \: we \: need \: to \: find \: x.[/tex]

[tex] \sf \: x + 3 = 45 \\ \sf \: x = 45 - 3 \\ \sf \: x = \underline{42}[/tex]

Answer ⟶ [tex]\boxed{\bf{D.42}}[/tex]

WE AIN'T LEARN THIS IN CLASS I'M GONNA CRY ​

Answers

Answer:

3x-6 Its is B

Step-by-step explanation:

We know that x is an even number. we have 2 even numbers which are consecutive. For example. If x was 6. the 2 before numbers would be 4 and 2. So the formula is x+(x-2)+(x-4). (-4)+(-2)=-6. In total there are 3x. So the answer would be 3x-6. Hope this helped ^^.

what multiplies to get 56 and adds to get 15

Answers

Answer:

8 & 7

Step-by-step explanation:

x*y = 56

x+y = 15

y = 15-x

x(15-x)=56

15x-x^2 = 56

x^2 -15x + 56 = 0

Factored Form:  y= (x-8)(x-7)    

What’s 4 divided by $5.32

Answers

Answer:

0.752

Step-by-step explanation:

Simplify the expression 5(x-3)(x^2+4x+1)

Answers

Simplify the expression 5(x-3)(x^2+4x+1)

it would be I think answer number 3

How many L of a 10% alcohol solution must be mixed with 9l of a 80% alcohol solution to make a 55% solution

Answers

Answer:

Step-by-step explanation:

The best thing to do for these solution/mixture problems is to make a table:

                          #L           *           %alcohol         =            L alcohol

    10% sol.

+   80% sol.                                                                                            

    55% sol.  

In this way, we can keep track of our information AND figure out the equation we need to use to solve it. Notice first, the equation along the top of the table:

#L * %alcohol = L alcohol  This tells us that we are multiplying the #L column times the %alcohol column to get the L alcohol column. Notice second, that there is a + sign to the far left, indicating that we are ADDING rows 1 and 2 together to get the mix. Let's start filling this in. The easy part is the % alcohol column. 10% alcohol has .10 alcohol as a decimal, likewise for the 80% and 55%:

                          #L             *            %alcohol          =         L alcohol

   10% sol.                                             .10

+  80% sol.                                            .80               =                          

   55% sol.                                            .55

Now to fill in the first column. We know that our unknown, from the problem, is the number of Liters, #L in the 10% solution, so that is x, and we also know that we have 9 L of the 80% alcohol. Filling that in:

                             #L            *             %alcohol           =         L alcohol

  10% sol.                x            *                .10

+ 80% sol.               9           *                 .80                                                

  55%                                                      .55

Now look back at the + sign. We are told that we are mixing the 10% with the 80%, so we are adding them together. So let's do that. We will also follow the rule for the table and multiply the first column times the second column to fill in the last column to complete the table:

                          #L           *             %alcohol            =              L alcohol

   10% sol.           x            *                 .10                  =                 .10x

+  80% sol.          9           *                 .80                  =                  7.2          

  55% sol.      (9 + x)         *                 .55                  =         .55x + 4.95

If we add the 2 solutions together to get the new solution in the first column, we will also add the L alcohol in the last column to get our equation:

.10x + 7.2 = .55x + 4.95 and

- .45x = - 2.25 so

x = 5L

We need 5 Liters of the 10% solution if we want to mix that with 9L of 80% solution to get 14 L of 55% solution.

Plz explain and I'll give brainly!!!

For this exponential function,
what is the output value (y),
when the input value (x) is 2?


y = 10.5x



Answers

Answer:

(2,21)

Step-by-step explanation:

y = 10.5x

plug 2 for x  in the equation

y = 10.5(2)

then multiply 2 by 10.5

y=21

[tex]\frac{x+4}{2} -\frac{3}{x-3} =1[/tex]

Quadratics null factor the given answer is x= 4,−3 but I don't know how to work it out

Answers

Follow each step in the picture

Answer:

Hello,

Step-by-step explanation:

[tex]x\neq 3\\\\\dfrac{x+4}{2} -\dfrac{3}{x-3} =1\\\\\dfrac{(x+4)(x-3)-2*3-2(x-3)}{2(x-3)} =0\\\\x^2+4x-3x-12-6-2x+6=0\\\\x^2-x-12=0\\\\x^2-4x+3x-12=0\\\\x(x-4)+3(x-4)=0\\\\(x-4)(x+3)=0\\\\x=4\ or\ x=-3\\[/tex]

Plz someone do this for me I’ll give a lot of points

Answers

Answer:

RW is congruent to WX

Step-by-step explanation:

We know that PW and WV and Angle W is vertical angle so We need to include that RW congruent to WX.

Findthe area of a quadrilateral ABCD in which AB=3cm,BC=4cm,CD=4cm,DA=5cm, and AC=5cm

Answers

Answer:

21cm

Step-by-step explanation:

3+4+4+5+5

I so

Which shows a difference of squares?10 y squared minus 4 x squared16 y squared minus x squared8 x squared minus 40 x 2564 x squared minus 48 x 9

Answers

Answer:

16 y squared minus x squared

Step-by-step explanation:

The other all expressions except 16y²-x² doesn't complete the square and its difference. Then we can check 16y²-x²,

Now take the expression 16y²-x² and we can try to solve it.

We can simplify the terms separately to associate them as perfect squares,

→ 16y² = (4y)² --- (1)

→ x² = (1x)² --- (2)

Now we can take the difference of eq. (1) and eq. (2),

→ 16y²-x²

→ (4y)²-(1x)² --- (3)

Now eq. (3) shows a difference of squares.

Another representation is,

→ (4y - x) (4y + x)

→ 16y²- x²

→ (4y)²- (1x)²

Therefore, the expression 16y²-x² shows a correct difference of squares.

Answer:

its b trust

Step-by-step explanation:

Which is the graph of f(x) = (x - 1)(x + 4)?
6
4
2
2
22
2
4
6
2
B42
2
O
O
4
2
2

Answers

Answer:

Graph D

Step-by-step explanation:

f(x)=(x-1)(x+4

roots x=1,-4,

x=0, y=-4

OAmalOHopeO

A box contains 6 red, 3 white, 2 green, and 1 black (in total 12) identical balls. What is the least number of balls necessary to take out randomly (without looking) to be sure of getting at least two white balls?

Answers

Answer:

Pick at least 4 balls to be sure that you are getting balls with the same color.

Step-by-step explanation:

Here, you want to know the least number of balls to be taken out of the box to be sure that you have all the three colors represented.

You know there are 12 identical balls, with the least numbers of balls being 1 and 2. Hence, to be able to know you have all the colors of balls represented, you will need to have taken all the less represented ones i.e the 1 and 2 , and this means that the next number of ball which would be taken will confidently confirm that you have taken all the colors since you would have exhausted picking other balls at this point.

So you shall be needing at least 4 balls picked to ensure that you have all the colors represented

PLEASE HELP!!
A 16 ft ladder is propped up against a building at an angle of 52°. How far up the wall does the ladder go?
a. 20.3 ft
b. 9.9 ft
c. 25.9 ft
d. 12.6 ft

Answers

Answer:

The correct answer is a) 12.6 ft

Step-by-step explanation:

I took the test and got it right:)

The branch of mathematics deals with the relations of the sides and angles of triangles and with the relevant functions of any angles. The ladder goes at a height of 12.6082 feet.

What is trigonometry?

The branch of mathematics deals with the relations of the sides and angles of triangles and with the relevant functions of any angles.

Given that the length of the Ladder is 16 feet while the angle between the ladder leaning against the wall is 52°.

Trigonometric Ratios are the values of all the trigonometric functions based on the value of the ratio of sides in a right-angled triangle.

Using Trigonometric Ratios of sin because of opposite over hypotenuse,

sin ∅=perpendicular/hypotenuse

sin(52) = x/16

16sin(52) = x

By calculating,

x = 12.6082 feet

Hence, the ladder goes at a height of 12.6082 feet.

Learn more about Trigonometry here:

https://brainly.com/question/26719838

#SPJ2

Grade improvement and study motivation

Answers

Answer:

yes because we are leraning new things

XYZ medication 250 mg , 2 tabs qid x 14 days. How many tablets will be dispensed from the pharmacy?

Answers

Answer:

112

Step-by-step explanation:

qid means 4 times a day. So 2 x 4 is 8. Then multiply that by 14 and you get 112.

112 tablets will be dispensed from the pharmacy such that [tex]XYZ[/tex] medication [tex]250[/tex] mg, [tex]2[/tex] tabs qid for [tex]14[/tex] days.

Unitary Method

The unitary method is a process to find the value of multiple units from the value of a single unit.

How to determine the number of tablets dispensed?

[tex]2[/tex] tabs qid means [tex](2[/tex] × [tex]4)[/tex] [tex]= 8[/tex] tabs per day will be required as per prescribed medication.

So, in [tex]14[/tex] days, the number of tabs required [tex]= 8[/tex] ×[tex]14 = 112[/tex].

Thus, 112 tablets will be dispensed from the pharmacy.

Learn more about unitary method here- https://brainly.com/question/22056199

#SPJ2

Angle ADB and CD are straight lines.

angle ADC = 5 x angle CDB

Work out the size of angle ADC.

Answers

Answer:

150 degrees

Step-by-step explanation:

5 + 1 = 6

180 : 6 = 30

30 x 5 = 150

Karma was given some birthday money. She puts money in an account. Every month after that, she deposits the same amount of money. The equation that models this situation is y=75x+60, where y is the amount of money in the account and x is the number of deposits. What does the y-intercept mean in this situation

Answers

Answer:

y represents the amount in the bank account after 7 months.

Step-by-step explanation:

Let karma gets x money on his birthday.

He has 60 in his account initially

He deposits teh same money for the seven months

So, the money he has after the seven months is 7 x + 60  

According to the question,

y =7 x + 60

It shows that the amount after seven months is y.  

10)
Ka
The longer base of a trapezoid has endpoints of (-2,- 4) and (4,0). The shorter base contains the point (3, 1).
(40)
(2)
We an equation of the line that contains the shorter base of the trapezoid. Provide valid mathematical reasoning and calculations to
you derived your equaiton.
of a trapezoid are parallel
Suation and explanation in the space provided.
Math symbols
Relations
Geometry

Answers

Answer:

y = 2·x/3 - 1

Step-by-step explanation:

The endpoints of the longer base of the trapezoid = (-2, -4), and (4, 0)

The point contained on the shorter base = (3, 1)

The slope of the line representing the longer base, m, is given as follows;

m = (0 - (-4))/(4 - (-2)) = 2/3

The slope of the shorter base = The slope of the longer base = m = 2/3

The equation of the line representing the shorter base in point and slope form is therefore;

y - 1 = (2/3)·(x - 3)

∴ y = 2·x/3 - 2 + 1 = 2·x/3 - 1

The equation of the shorter side of the trapezoid is y = 2·x/3 - 1


What is the domain of the function on the graph?

Answers

Answer:

Step-by-step explanation:

Domain covers x values only (where range covers y values only). Domains are stated from the absolute lowest x value included on the graph to the highest. Looking at our graph, we have a solid dot at x = -3, and there is no part of the graph that goes to the left of -3. So -3 is the absolute lowest x value on the graph. When you follow the blue graph, you notice that it goes off the graph "paper" and does not have a solid dot at the tail end of it. This means that the graph will go on forever. This implies infinity, which looks like this: ∞

The lowest x value is -3 and the highest is ∞ so there are a couple of ways in which you can state this:

D = {x | -3 ≤ x < ∞} or D = {x ≥ -3} Either one will work

5x+y=2 4x+y=4
how do i solve this?

Answers

[tex]\boxed{\large{\bold{\blue{ANSWER~:) }}}}[/tex]

See this attachment

[tex]\boxed{\boxed{\sf{x=-2~and~y=12 }}}[/tex]

Question 4 of 25
Suppose a normal distribution has a mean of 62 and a standard deviation of
4. What is the probability that a data value is between 58 and 64? Round your
answer to the nearest tenth of a percent.
A. 53.3%
B. 54.3%
C. 52.3%
D. 51.3%

Answers

Answer:

Step-by-step explanation:

We are looking for P(58 < x < 64). We need to find the percentage to the left of the z-scores for each of these numbers. To find the z scores, use the formula:

[tex]z=\frac{x_i-\bar{x}}{\sigma}[/tex]

[tex]z=\frac{58-62}{4}[/tex] which gives us a z-score of -1. The percentage of numbers to the left of a z-score of -1 is .1586553

Now for the other z-score:

[tex]z=\frac{64-64}{4}[/tex] which gives us a z-score of .5. The percentage of numbers to the left of a z-score of .5 is .69146246

The lower percentage subtracted from the higher gives the area in question:

.69146246 - .1586553 = .53280716, or as a percentage, 53.3%, choice A.

Find the length of side x in simplest radical form with a rational denominator.
60°
12
Submit Answer
Answer: =
attempt 1 out of 2
PLSSSSS HELP

Answers

Answer:

24

Step-by-step explanation:

cos 60 = adjacent / hypotenuse

1 / 2 = 12 / x

x = 2 ( 12 )

x = 24

Someone, please help! Thank you!
The ratio of measures of angles of a polygon is 3:1:4:1:5:9:2. What is the measure of the largest angle?

Answers

Answer:

324°

Step-by-step explanation:

Totak angles in a 7 sided polygon = 900°

Side A = 3

Side B = 1

Side C = 4

Side D = 1

Side E = 5

Side F = 9

Side G = 2

Total = 25

The largest angle has the largest ratio which is side F

Angle in side F = Ratio of side F / total ratio × 900°

= 9/25 × 900°

= 0.36 × 900°

= 324°

The largest angle = 324°

The smallest angles are angles with the lowest ratio which are side B and side D

Angle in side B = Ratio of side F / total ratio × 900°

= 1/25 × 900°

= 0.04 × 900°

= 36°

The smallest angles are angles 36°

Complete the sentence. The amount of time it takes to drive from your house to the library is most likely to be a function of the _____.

Answers

Answer:

D. distance

Step-by-step explanation:

really ? given the choices you needed help with that ?

only D makes sense. the rest is completely ridiculous and has nothing to do with driving from the house to the library.

if V = 1/3 BH, what is h expressed in terms of B and V?
A) 1/3VB
B) V/3B
C) 3V/B
D) 3VB

Answers

V=(1/3)(B)(h)

multiply both sides by 3

3V=Bh

divide both sides by B

3V/B=h

someone please help me

Answers

Step-by-step explanation:

D) (A³/2)³ = A⁹/8

A³ whole to the power 3 is A⁹

2³ is 8

Question 2 of 5
Drag the tiles to the correct boxes to complete the pairs. Not all tiles will be used.
Match each explicit formula to its corresponding recursive formula,

Answers

Given:

The different recursive formulae.

To find:

The explicit formulae for the given recursive formulae.

Solution:

The recursive formula of an arithmetic sequence is [tex]f(n)=f(n-1)+d, f(1)=a,n\geq 2[/tex] and the explicit formula is [tex]f(n)=a+(n-1)d[/tex], where a is the first term and d is the common difference.

The recursive formula of a geometric sequence is [tex]f(n)=rf(n-1), f(1)=a,n\geq 2[/tex] and the explicit formula is [tex]f(n)=ar^{n-1}[/tex], where a is the first term and r is the common ratio.

The first recursive formula is:

[tex]f(1)=5[/tex]

[tex]f(n)=f(n-1)+5[/tex] for [tex]n\geq 2[/tex].

It is the recursive formula of an arithmetic sequence with first term 5 and common difference 5. So, the explicit formula for this recursive formula is:

[tex]f(n)=5+(n-1)(5)[/tex]

[tex]f(n)=5+5(n-1)[/tex]

Therefore, the correct option is A, i.e., [tex]f(n)=5+5(n-1)[/tex].

The second recursive formula is:

[tex]f(1)=5[/tex]

[tex]f(n)=3f(n-1)[/tex] for [tex]n\geq 2[/tex].

It is the recursive formula of a geometric sequence with first term 5 and common ratio 3. So, the explicit formula for this recursive formula is:

[tex]f(n)=5(3)^{n-1}[/tex]

Therefore, the correct option is F, i.e., [tex]f(n)=5(3)^{n-1}[/tex].

The third recursive formula is:

[tex]f(1)=5[/tex]

[tex]f(n)=f(n-1)+3[/tex] for [tex]n\geq 2[/tex].

It is the recursive formula of an arithmetic sequence with first term 5 and common difference 3. So, the explicit formula for this recursive formula is:

[tex]f(n)=5+(n-1)(3)[/tex]

[tex]f(n)=5+3(n-1)[/tex]

Therefore, the correct option is D, i.e., [tex]f(n)=5+3(n-1)[/tex].

Answer:

From Edmentum :)

Other Questions
Please help , write your answer I will be giving 10 points PLEASE HELPPPP I ONLY HAVE 3 MIN LEFTTTTT I NEED TO PASS THIS TEST f an egg can move, what provides the energy for movement?Question 7 options:The uterus provides the energy for the movement of the cilia and fimbriae. The ovum itself is motile.Fructose provides the energy for the movement of the cilia and fimbriae. The ovum itself is non-motile.ATP provides the energy for the movement egg since the ovum itself is motile and self reliant.ATP provides the energy for the movement of the cilia and fimbriae. The ovum itself is non-motile. If 4 over 7 ton of concrete covers 7 over 8 of a bridge, how many tons of concrete are required to cover the entire bridge? Can someone help me on 6? verify a(b-c)=ab-ac for a=1.6;b=1/-2;& c=-5/-7 What is the biggest concern many people have with the recognition of places like Kosovo as sovereign nations and how it may affect Canada and other nations? Which of the following choices reflect the change in the miller's daughter from the beginning to the end of the story? She changes from helpless to powerful She changes from humble to greedy She changes from selfish to generous. She changes from sad to happy (Question is in the image)please I really need help Changing some of the values in a data set is called transforming the data.A. TrueB. False If the product of two numbers is 1800 and its H.C.F. Is 30, find the L.C.M. Please help, Im not sure about this question. My class consists of 8 men and 7 women. I want to pick a group of 6 people for research.Write each answer using fraction as needed.a. In how many different ways can I pick this group?b. What is the probability of having exactly 3 men in the group?c. What is the probability of all the selected people in group are women?d. What is the probability of having at least one man in the group? Your father has travelled abroad and hardly knowshow you are getting on in class.writea letter to inform tum about your progressat school Denver's elevation is 5280 feet above sea level. Death Valley is -282 feet. Is Death Valley located above sea level or below sea level???(plz answer, due date is semtemper) what are the qualities of woollen fabric that makes it suitable for winter I'm so excited to be apart of the team organizing our school's annual 5k!Find the error What is the surface area of a sphere with a diameter of 14 cm? Question 6 options:175.84 cm^2351.68 cm^2615.44 cm^22461.76 cm^2Show your work: please help me with this The following sentences have just one part of speech missing. Can you tell which one it is?1. Well, I looked in the car and I couldnt find the purple sweater.2. Wow! You and little John swim well, but I dont.3. Oh, they are happily baking cookies and brownies in the kitchen.